Using root test and ratio test for divergence

Click For Summary
The series in question is n(-3)^(n+1) / 4^(n-1), and the discussion revolves around determining its convergence or divergence using the ratio and root tests. The ratio test was attempted but did not simplify easily, leading to confusion about applying L'Hospital's rule. A participant pointed out that the ratio simplifies to -3, making L'Hospital's rule unnecessary. The root test also posed challenges, particularly with the limit of (n-1)/n as n approaches infinity. The conversation highlights the importance of recognizing simplifications in series tests for convergence analysis.
superdave
Messages
149
Reaction score
3

Homework Statement



Does this series converge or diverge?

Series from n=1 to infinity n(-3)^(n+1) / 4^(n-1)



Homework Equations





The Attempt at a Solution



Okay, I've tried it both ways.

Ratio test:

lim n --> inf. ((n+1)*(-3)^(n+1)/4^n) / (n * (-3)^n / 4^(n-1))

Now, that doesn't appear to simplify in anyway that would make using l'hospital's rule possible to find the limit.

Root test:

lim n --> inf. of -3*n^(1/n) / 4^((n-1)/n)

That bottom part throws me off.
 
Physics news on Phys.org
Why does it through you off? What is the limit of (n-1)/n as n goes to infinity?
 
superdave said:

Homework Statement



Does this series converge or diverge?

Series from n=1 to infinity n(-3)^(n+1) / 4^(n-1)



Homework Equations





The Attempt at a Solution



Okay, I've tried it both ways.

Ratio test:

lim n --> inf. ((n+1)*(-3)^(n+1)/4^n) / (n * (-3)^n / 4^(n-1))

Now, that doesn't appear to simplify in anyway that would make using l'hospital's rule possible to find the limit.
Seriously? You are aware, are you not, that (-3)^(n+1)/(-3)^n= -3? The (4^n part is just as easy! You should not need L'Hospital's rule.

Root test:

lim n --> inf. of -3*n^(1/n) / 4^((n-1)/n)

That bottom part throws me off.
 
Question: A clock's minute hand has length 4 and its hour hand has length 3. What is the distance between the tips at the moment when it is increasing most rapidly?(Putnam Exam Question) Answer: Making assumption that both the hands moves at constant angular velocities, the answer is ## \sqrt{7} .## But don't you think this assumption is somewhat doubtful and wrong?

Similar threads

  • · Replies 4 ·
Replies
4
Views
1K
  • · Replies 2 ·
Replies
2
Views
2K
  • · Replies 2 ·
Replies
2
Views
2K
  • · Replies 6 ·
Replies
6
Views
2K
  • · Replies 1 ·
Replies
1
Views
2K
  • · Replies 7 ·
Replies
7
Views
2K
Replies
7
Views
2K
  • · Replies 2 ·
Replies
2
Views
1K
  • · Replies 3 ·
Replies
3
Views
2K
  • · Replies 6 ·
Replies
6
Views
2K